Inflation!

This topic has expert replies
User avatar
Legendary Member
Posts: 1083
Joined: Wed Feb 24, 2010 9:38 pm
Thanked: 127 times
Followed by:14 members

Inflation!

by gmat_perfect » Sat Aug 28, 2010 12:33 pm
Last year the rate of inflation was 1.2 percent, but during the current year it has been 4 percent. We can conclude that inflation is on an upward trend and the rate will be still higher next year.

Which of the following, if true most seriously weakens the conclusion above?

A. The inflation figures were computed on the basis of a representative sample of economic data rather than all of the available data.
B. Last year a dip in oil prices brought inflation temporarily below its recent stable annual level of 4 percent.
C. Increases in the pay of some workers are tied to the level of inflation, and at an inflaction rate of 4 percent or above, these pay raises constitute a force causing further inflation.
D. The 1.2 percent rate of inflation last year represented a 10 year low.
E. Government intervention cannot affect the rate of inflation to any significant degree.

[spoiler]OA: E[/spoiler]

Why is D incorrect? I want to know the exactly why D is incorrect.

Please explain.

Thanks.

User avatar
Legendary Member
Posts: 1172
Joined: Wed Apr 28, 2010 6:20 pm
Thanked: 74 times
Followed by:4 members

by uwhusky » Sat Aug 28, 2010 12:42 pm
The conclusion is:

"We can conclude that inflation is on an upward trend and the rate will be still higher next year."

So whether last year is 10 year low is completely irrelevant to the conclusion that the inflation will continue to rise.

What was your reasoning why D is a contender?

User avatar
Legendary Member
Posts: 1083
Joined: Wed Feb 24, 2010 9:38 pm
Thanked: 127 times
Followed by:14 members

by gmat_perfect » Sat Aug 28, 2010 1:43 pm
uwhusky wrote:The conclusion is:

"We can conclude that inflation is on an upward trend and the rate will be still higher next year."

So whether last year is 10 year low is completely irrelevant to the conclusion that the inflation will continue to rise.

What was your reasoning why D is a contender?
We need to weaken the conclusion, and the conclusion is "We can conclude that inflation is on an upward trend and the rate will be still higher next year"

--> Inflation is on an upward. trend.

if we can not conclude from a one year data as it is given in D, then how can we conclude that the inflation will be upward?

it was my reasoning.

Would you please explain in more detail?

Thanks.

User avatar
Legendary Member
Posts: 1172
Joined: Wed Apr 28, 2010 6:20 pm
Thanked: 74 times
Followed by:4 members

by uwhusky » Sat Aug 28, 2010 4:20 pm
I am confused as to how you are reasoning D. What D does is giving a label to the level of inflation last year, but what it doesn't do is offering any explanation or establish a scenario that may be viewed as strengthening or weakening the conclusion.

Btw, I think your OA is incorrect.

Legendary Member
Posts: 1119
Joined: Fri May 07, 2010 8:50 am
Thanked: 29 times
Followed by:3 members

by diebeatsthegmat » Sat Aug 28, 2010 5:48 pm
gmat_perfect wrote:Last year the rate of inflation was 1.2 percent, but during the current year it has been 4 percent. We can conclude that inflation is on an upward trend and the rate will be still higher next year.

Which of the following, if true most seriously weakens the conclusion above?

A. The inflation figures were computed on the basis of a representative sample of economic data rather than all of the available data.
B. Last year a dip in oil prices brought inflation temporarily below its recent stable annual level of 4 percent.
C. Increases in the pay of some workers are tied to the level of inflation, and at an inflaction rate of 4 percent or above, these pay raises constitute a force causing further inflation.
D. The 1.2 percent rate of inflation last year represented a 10 year low.
E. Government intervention cannot affect the rate of inflation to any significant degree.

[spoiler]OA: E[/spoiler]

Why is D incorrect? I want to know the exactly why D is incorrect.


Please explain.

Thanks.

are you sure the answer is E? because acorrding to me, and according to what i remember this CR is quite "popular", i've seen
it in OG 10 and 11 and review OG book and the answer must be B
it gave another reason explained why the inflation last year decreased
D is not relevant

Legendary Member
Posts: 995
Joined: Tue Apr 13, 2010 11:56 pm
Thanked: 31 times
Followed by:1 members